[Date Prev][Date Next][Thread Prev][Thread Next][Date Index][Thread Index]

Re: [obm-l] Re: Fatorial <> Quadrado



Pensei um pouco nesse problema e, sei lá porque que razão, parei pra contar
quantos "dois" aparecem nas fatorações de números (pares) consecutivos.

Encontrei a seguinte sequência:

1 (2 contém exatamente um 2)
2 (4 contém dois 2...)
1
3 (8 tem 3, deu pra entender né)
1
2
1
4
1
2
1
3
1
2
1
5
(...)

Era de se esperar que aparecessem simetrias, mas confesso que me surpreendi
em constatar que as somas parciais dessa sequencia nos blocos terminados em
posições 2^n são todas forma (2^n) -1 !!
Ex:
Somando até 4:
1+2 = 3

Somando até 8:
1+2+1+3 = 7

Somando até 32:
1+2+1+3+1+2+1+4+1+2+1+3+1+2+1+5 = 31

Depois do choque, vi que o fato é não só razoável como também algo esperado,
já que entre 1 e 2^n vão haver 2^(n-1) múltiplos de 2, 2^(n-2) múltiplos de
4, 2^(n-3) múltiplos de 8 e assim por diante. Escrevendo essa soma em
binário (isso te lembra alguma coisa, Pina ?) vamos tem um cara da forma
(11111111....)base2 onde aparecem n 1´s , o que é justamente algo do tipo
2^n -1 !!

Bom, isso não me levou a concluir nada sobre fatoriais e quadrados, mas
achei válido mandar pra lista assim mesmo :-))

Saudações
Will



----- Original Message -----
From: "Claudio Buffara" <claudio.buffara@terra.com.br>
To: "Lista OBM" <obm-l@mat.puc-rio.br>
Sent: Wednesday, September 17, 2003 9:24 PM
Subject: [obm-l] Re: Fatorial <> Quadrado


on 16.09.03 16:46, Claudio Buffara at claudio.buffara@terra.com.br wrote:

> Oi, pessoal:
>
> Alguem conhece alguma demonstracao de que nenhum fatorial > 1 eh quadrado
> perfeito que nao use o postulado de Bertrand?
>
> Um abraco,
> Claudio.

O que eu acho estranho eh que a demonstracao do postulado de Bertrand (pelo
menos a que eu conheco) baseia-se numa analise dos fatores primos de
Binom(2n,n) = (2n)!/n!^2. Assim, seria de se esperar que uma analise dos
fatores primos de n! fosse mais simples do que a dos fatores de Binom(2n,n)
e, portanto, que existisse uma demonstracao do resultado acima que nao
envolvesse o postulado de Bertrand.

Eh fato (decorrente do postulado de Bertrand) que se p eh o maior primo <=
n, entao n < 2p e, portanto, o expoente de p em n! eh 1, o que impede que n!
seja um quadrado perfeito.

O problema eh que sem Bertrand eu nao consigo provar que n < 2p, ou seja,
que a situacao em que os numeros: p+1, p+2, ..., 2p-1, 2p, ..., n (n >= 2p)
sao todos compostos nunca ocorre.

Um abraco,
Claudio.

=========================================================================
Instruções para entrar na lista, sair da lista e usar a lista em
http://www.mat.puc-rio.br/~nicolau/olimp/obm-l.html
=========================================================================


=========================================================================
Instruções para entrar na lista, sair da lista e usar a lista em
http://www.mat.puc-rio.br/~nicolau/olimp/obm-l.html
=========================================================================